Math, asked by gurpreers5110, 4 months ago

lf HCF of two numbers be 40 then which of the following cannot be their LCM.​

Answers

Answered by ayushstudy007
0

Answer:

1st no bro i think so try

Answered by rajanshu26
4

Step-by-step explanation:

tell the optionss........

Similar questions